LSAT and Law School Admissions Forum

Get expert LSAT preparation and law school admissions advice from PowerScore Test Preparation.

 Administrator
PowerScore Staff
  • PowerScore Staff
  • Posts: 8916
  • Joined: Feb 02, 2011
|
#35121
Complete Question Explanation

Must Be True—CE. The correct answer choice is (E)

Your task in this Must Be True question is to select the answer choice that contains either a
restatement of one of the facts in the stimulus, or an inference permissible from a combination of the
facts.

..... Fact: ..... the calm, shallow waters of coastal estuaries are easily polluted by nutrientrich
..... ..... ..... sewage

..... Fact: ..... when estuary waters become overnutrified by the sewage, algae proliferate

..... Fact: ..... abundant algae (i.e., when algae proliferate), in turn, sometimes provide a rich
..... ..... ..... food source for microorganisms that are toxic to fish, killing most of the fish
..... ..... ..... in the estuary

Your prephrase is that an inference supported by this fact set is that if a coastal estuary is polluted
by nutrient-rich sewage, most of the fish in the estuary may be killed as a result. The correct answer
likely will test you on this inference, though it may instead test you on a restatement of any fact in
the stimulus.

The incorrect answers will contain information that is not supported by the stimulus.

Answer choice (A): This stimulus did not mention the comparative likelihood of fish death resulting
from various types of pollution.

Answer choice (B): While the stimulus stated that the abundant algae provide a rich food source for
this microorganisms, the facts do not support the inference that they reproduce more quickly than
other types of microorganisms.

Answer choice (C): This answer choice is improperly restrictive. Citing one source of harm to the
fish without mentioning others does not preclude the existence of other threats to the fish.

Answer choice (D): This is a Mistaken Reversal of the conditional relationship provided in the
second sentence of the stimulus.

Answer choice (E): This is the correct answer choice. This answer reflects the inference
permissible from a combination of the facts in the stimulus, discussed above.
 lunsandy
  • Posts: 61
  • Joined: Oct 14, 2017
|
#41814
Hi Powerscore!

I read this question completely wrong when I was doing the exam and thought of it as an assumption question rather than a MBT/MSS question. Would C be correct if the question was an assumption question? Upon review, I see E is the correct answer because of "can," which follows from the stim. "sometimes" and our conclusion "most."

Thanks a lot!
 Jennifer Janowsky
PowerScore Staff
  • PowerScore Staff
  • Posts: 90
  • Joined: Aug 20, 2017
|
#42046
Hi! I don't believe that (C) is an assumption that must be made in this stimulus. It should not have made much of a difference to the content of the stimulus if there were other sources of harm from pollution, as it states as a fact that the toxicity kills most of the fish.
User avatar
 SGD2021
  • Posts: 72
  • Joined: Nov 01, 2021
|
#91948
Hello,

Can you please explain why Answer Choice D is a mistaken reversal of the conditional relationship provided in the second sentence of the stimulus? What are some tips or questions that we can ask ourselves to determine which is the sufficient and which is the necessary condition in a statement like this (Algae will not proliferate in coastal estuaries that are not polluted by nutrient-rich sewage) that does not have sufficient/necessary condition indicators?
User avatar
 atierney
PowerScore Staff
  • PowerScore Staff
  • Posts: 215
  • Joined: Jul 06, 2021
|
#92000
Hello,

Yes, so, IF overnutrified, THEN algae proliferation, but NOT IF not overnutrified, THEN not algae proliferation. This is essentially a mistaken negation.

Generally speaking, the conclusory component of any statement is going to be the necessary condition. For example, I will not answer any question that has already been answered. What part of that statement is the conclusory fact? It is when I will not answer a question. And which questions, or under what conditions, will I not answer a question? It is to those that have already been answered. So, IF a question has already been answered, THEN what can we necessarily conclude? I won't answer the question!

Now, applying this to the question at hand (in the answer choice), when is it or under what conditions sufficient will algae not proliferate? When the coastal estuaries are not polluted by sewage! IF not polluted by sewage, what can we necessarily conclude? Algae will not proliferate, i.e., the necessary condition.

Hopefully, this helps answer your question. Do let me know if you have follow up unanswered questions, i.e., IF you have follow up questions, THEN let me know!

Get the most out of your LSAT Prep Plus subscription.

Analyze and track your performance with our Testing and Analytics Package.